Side 9 av 11

Re: Ulikhetmaraton

Lagt inn: 31/10-2018 13:37
av Gustav
Markus skrev:Jeg mistenker at problemet, i likhet med mye annet, kan også løses på en annen måte. Med for eksempel Cauchy-Schwarz kan vi også finne en nedre grense for uttrykket; $(n+\frac12)^2 \geq \left (\sum_{i=1}^n a_i \right) \left ( \sum_{i=1}^n \frac{1}{a_i} \right) \geq n^2$ Den øvre og nedre grensen er så nærme hverandre, at jeg nesten tror det er med vilje, men jeg ser ikke helt veien videre herifra. Hvis noen vil hjelpe til eller gi hint settes det stor pris på!

Forresten, zzzivert, hvorfor kan du av homogenitet anta at $a_1=\frac12$?
Hint : WLOG anta at $a_1$ er minst og $a_2$ størst. Bruk Cauchy-Schwarz på

$$ \left [ (a_1+a_2)+a_3+\cdots + a_n\right ]\left [(\frac{1}{a_1}+\frac{1}{a_2})+\frac{1}{a_3}+\cdots +\frac{1}{a_n}\right ]$$

Angående det siste spørsmålet: Siden ulikheten er homogen er den ekvivalent med om du multipliserer alle $a_i$ med en skalar $k$, dermed kan du WLOG anta at $a_1=\frac12$ (velg $k=\frac{1}{2a_1}$).

Re: Ulikhetmaraton

Lagt inn: 01/11-2018 17:53
av Gustav
Oppfølger: Vis at for alle positive heltall $n$ gjelder $\sqrt{2}\cdot \sqrt[4]{4}\cdot \sqrt[8]{8}\cdots \sqrt[2^n]{2^n}\le n+1$.

Re: Ulikhetmaraton

Lagt inn: 05/11-2018 20:29
av zzzivert
La [tex]p(n)=\sqrt{2}\cdot \sqrt[4]{4}\cdot \sqrt[8]{8}\cdots \sqrt[2^n]{2^n}[/tex]. Da har vi:
[tex]p(n)=2^{s(n)}[/tex] der [tex]s(n)=\frac{1}{2}+\frac{2}{4}+\frac{3}{8}+\cdots +\frac{n}{2^n}[/tex].

La $f(x)=1+x+x^2+\cdots +x^n=\frac{1-x^{n+1}}{1-x}$. Da er:
$f^{\prime}(x)=1+2x+3x^2+\cdots + nx^{n-1} \\
=(\frac{1-x^{n+1}}{1-x})^{\prime}=\frac{-(n+1)x^n(1-x)+1-x^{n+1}}{(1-x)^2} \\
=\frac{nx^{n+1}-(n+1)x^n+1}{(1-x)^2}$

Vi ser at $s(n)=\frac{1}{2}f^{\prime}(\frac{1}{2})=2-(n+2)(\frac{1}{2})^n<2$
Derfor er $p(n)<4$ og det holder å sjekke at
$p(n)\le n+1$ for $n=1,2$, som stemmer.

Re: Ulikhetmaraton

Lagt inn: 05/11-2018 20:32
av zzzivert
Oppfølger:

La $a,b,c$ være positive reelle tall s.a. $a+b+c=1$. Vis at
$\sqrt{ab+c}+\sqrt{bc+a}+\sqrt{ca+b}\ge 1+\sqrt{ab}+\sqrt{bc}+\sqrt{ca}$

Re: Ulikhetmaraton

Lagt inn: 10/11-2018 03:02
av Gustav
zzzivert skrev:La [tex]p(n)=\sqrt{2}\cdot \sqrt[4]{4}\cdot \sqrt[8]{8}\cdots \sqrt[2^n]{2^n}[/tex]. Da har vi:
[tex]p(n)=2^{s(n)}[/tex] der [tex]s(n)=\frac{1}{2}+\frac{2}{4}+\frac{3}{8}+\cdots +\frac{n}{2^n}[/tex].

La $f(x)=1+x+x^2+\cdots +x^n=\frac{1-x^{n+1}}{1-x}$. Da er:
$f^{\prime}(x)=1+2x+3x^2+\cdots + nx^{n-1} \\
=(\frac{1-x^{n+1}}{1-x})^{\prime}=\frac{-(n+1)x^n(1-x)+1-x^{n+1}}{(1-x)^2} \\
=\frac{nx^{n+1}-(n+1)x^n+1}{(1-x)^2}$

Vi ser at $s(n)=\frac{1}{2}f^{\prime}(\frac{1}{2})=2-(n+2)(\frac{1}{2})^n<2$
Derfor er $p(n)<4$ og det holder å sjekke at
$p(n)\le n+1$ for $n=1,2$, som stemmer.
Flott :D

Re: Ulikhetmaraton

Lagt inn: 18/11-2018 17:50
av Markus
Gustav skrev:Hint : WLOG anta at $a_1$ er minst og $a_2$ størst. Bruk Cauchy-Schwarz på

$$ \left [ (a_1+a_2)+a_3+\cdots + a_n\right ]\left [(\frac{1}{a_1}+\frac{1}{a_2})+\frac{1}{a_3}+\cdots +\frac{1}{a_n}\right ]$$
Tok litt tid å få denne til, men det var jo ikke så mye verre enn å faktisk skrive ut uttrykket ved å bruke hintet :oops:

Anta (wlog) at $a_1$ er minst og at $a_2$ er størst. Cauchy-Schwarz, sammen med den gitte bibetingelsen i oppgaven, gir $$\left(n+\frac12 \right)^2 \geq \left((a_1+a_2)+a_3+\dots+a_n \right)\left(\left(\frac{1}{a_1} + \frac{1}{a_2} \right) + \frac{1}{a_3} + \dots + \frac{1}{a_n} \right) \geq \left(\sqrt{2+\frac{a_2}{a_1} + \frac{a_1}{a_2}} + (n-2) \right)^2 \\ \implies \left(n+\frac12 \right)^2 \geq \left(\sqrt{2+\frac{a_2}{a_1} + \frac{a_1}{a_2}} + (n-2) \right)^2 $$ Siden $a_1,a_2$ er positive tall, kan vi ta roten på begge sidene, og flytte om litt for å få $$\frac{5}{2} \geq \sqrt{2 + \frac{a_2}{a_1}+\frac{a_1}{a_2}} $$ Siden $a_1$ er antatt minst og $a_2$ er antatt størst, kan vi wlog anta at $\frac{a_1}{a_2} \leq 1$, så $\sqrt{2+\frac{a_2}{a_1}+\frac{a_1}{a_2}} \geq \sqrt{3+\frac{a_2}{a_1}}$ Dette gir endelig at $$\frac{5}{2} \geq \sqrt{3+\frac{a_2}{a_1}} \implies \frac{13}{4}a_1 \geq a_2$$ Som er et sterkere resultat enn det vi skulle vise. Hvordan løste du den Gustav?

Re: Ulikhetmaraton

Lagt inn: 20/11-2018 00:59
av Gustav
Markus skrev: Siden $a_1$ er antatt minst og $a_2$ er antatt størst, kan vi wlog anta at $\frac{a_1}{a_2} \leq 1$, så $\sqrt{2+\frac{a_2}{a_1}+\frac{a_1}{a_2}} \geq \sqrt{3+\frac{a_2}{a_1}}$
Dette steget kan vel ikke stemme da den siste ulikheten er ekvivalent med $\frac{a_1}{a_2}\ge 1$.

Markus skrev:Siden $a_1,a_2$ er positive tall, kan vi ta roten på begge sidene, og flytte om litt for å få $$\frac{5}{2} \geq \sqrt{2 + \frac{a_2}{a_1}+\frac{a_1}{a_2}} $$ Siden $a_1$ er antatt minst og $a_2$ er antatt størst, kan vi wlog anta at $\frac{a_1}{a_2} \leq 1$,
Den første ulikheten er ekvivalent med

$(4a_1-a_2)(a_1-4a_2)\le 0$, så da følger at $4a_1\ge a_2$ som skulle vises.

Re: Ulikhetmaraton

Lagt inn: 20/11-2018 19:59
av Markus
Gustav skrev:
Markus skrev: Siden $a_1$ er antatt minst og $a_2$ er antatt størst, kan vi wlog anta at $\frac{a_1}{a_2} \leq 1$, så $\sqrt{2+\frac{a_2}{a_1}+\frac{a_1}{a_2}} \geq \sqrt{3+\frac{a_2}{a_1}}$
Dette steget kan vel ikke stemme da den siste ulikheten er ekvivalent med $\frac{a_1}{a_2}\ge 1$.

Markus skrev:Siden $a_1,a_2$ er positive tall, kan vi ta roten på begge sidene, og flytte om litt for å få $$\frac{5}{2} \geq \sqrt{2 + \frac{a_2}{a_1}+\frac{a_1}{a_2}} $$ Siden $a_1$ er antatt minst og $a_2$ er antatt størst, kan vi wlog anta at $\frac{a_1}{a_2} \leq 1$,
Den første ulikheten er ekvivalent med

$(4a_1-a_2)(a_1-4a_2)\le 0$, så da følger at $4a_1\ge a_2$ som skulle vises.
Ja herregud, den ene ulikheten der stemmer jo ikke i det hele tatt... Hm..

Vi er enige i $\frac{5}{2} \geq \sqrt{\frac{a_1}{a_2}+\frac{a_2}{a_1} + 2 }$? Hvis jeg forstår deg rett så kan vi gjøre litt algebra med denne og få $4a_1^2-13a_1a_2+4a_2^2 \geq 0 \implies 4a_1^2-17a_1a_2 + 4a_2^2 = (4a_1-a_2)(a_1-4a_2)\geq 0$? Jeg gjorde dette en gang til, altså startet med $$\frac{5}{2} \geq \sqrt{\frac{a_1}{a_2}+\frac{a_2}{a_1} + 2 }$$ også kom jeg hit $$\frac{13}{4} \geq \frac{a_1}{a_2} + \frac{a_2}{a_1}$$ Siden $\frac{a_1}{a_2}\geq 0$, fordi $a_1,a_2$ er positive, vil vi få $\frac{13}{4} \geq \frac{a_2}{a_1}$, men herifra følger den samme konklusjonen som jeg hadde istad, så jeg mistenker at jeg gjør en feil et sted, men jeg klarer ikke å se den.

Re: Ulikhetmaraton

Lagt inn: 20/11-2018 20:56
av Gustav
Markus skrev:
Vi er enige i $\frac{5}{2} \geq \sqrt{\frac{a_1}{a_2}+\frac{a_2}{a_1} + 2 }$? Hvis jeg forstår deg rett så kan vi gjøre litt algebra med denne og få $4a_1^2-13a_1a_2+4a_2^2 \geq 0 \implies 4a_1^2-17a_1a_2 + 4a_2^2 = (4a_1-a_2)(a_1-4a_2)\geq 0$? Jeg gjorde dette en gang til, altså startet med $$\frac{5}{2} \geq \sqrt{\frac{a_1}{a_2}+\frac{a_2}{a_1} + 2 }$$ også kom jeg hit $$\frac{13}{4} \geq \frac{a_1}{a_2} + \frac{a_2}{a_1}$$ Siden $\frac{a_1}{a_2}\geq 0$, fordi $a_1,a_2$ er positive, vil vi få $\frac{13}{4} \geq \frac{a_2}{a_1}$, men herifra følger den samme konklusjonen som jeg hadde istad, så jeg mistenker at jeg gjør en feil et sted, men jeg klarer ikke å se den.
Antar du har regna litt feil her. Det skal vel heller være

$$\frac{17}{4} \geq \frac{a_1}{a_2} + \frac{a_2}{a_1}$$, som er ekvivalent med $(4a_1-a_2)(a_1-4a_2)\leq 0$. Siden $a_2\ge a_1$ er $a_1-4a_2<0$, og vi kan konkludere med at $4a_1-a_2\ge 0$, altså at $4a_1\ge a_2$, som var det som skulle vises.

Re: Ulikhetmaraton

Lagt inn: 17/01-2019 01:27
av Gustav
La $a_0,a_1,a_2,\cdots$ være en uendelig følge av reelle tall slik at $\frac{a_{n-1}+a_{n+1}}{2}\ge a_n$ for alle positive heltall $n$. Vis at $$ \frac{a_0+a_{n+1}}{2}\ge \frac{a_1+a_2+ \cdots +a_n}{n}$$ holder for alle positive heltall $n$.

Re: Ulikhetmaraton

Lagt inn: 19/01-2019 13:40
av zzzivert
La $b_n=a_n-a_{n-1}$, da har vi:
$a_n=a_{n-1}+b_n=a_{n-2}+b_n+b_{n-1}=\cdots =a_0+\sum^n_{i=1}b_i$.
Da blir den gitte ulikheten:
$\frac{a_{n-1}+a_{n+1}}{2}\ge a_n$
$\frac{a_{n-1}+a_{n-1}+b_{n+1}+b_n}{2}\ge a_{n-1}+b_n$
$b_{n+1}\ge b_n$
Og ulikheten vi skal vise:
$\frac{a_0+a_{n+1}}{2}\ge \frac{a_1+a_2+\cdots + a_n}{n}$
$\frac{a_0+a_0+\sum^{n+1}_{i=1}b_i}{2}\ge \frac{(a_0+b_1)+(a_0+b_1+b_2)+\cdots +(a_0+\sum^n_{i=1}b_i)}{n}$
$\frac{1}{2}\sum^{n+1}_{i=1}b_i\ge \frac{nb_1+(n-1)b_2+\cdots+2b_{n-1}+b_n}{n}$
Vi har også ulikheten:
$\frac{1}{2}\sum^{n+1}_{i=1}b_i\ge \frac{n+1}{2n}\sum^{n}_{i=1}b_i$
siden det er ekvivalent med
$nb_{n+1}\ge \sum^{n}_{i=1}b_i$.
Derfor får vi:
$\frac{1}{2}\sum^{n+1}_{i=1}b_i\ge \frac{n+1}{2n}\sum^{n}_{i=1}b_i=\frac{\frac{n(n+1)}{2}}{n}\frac{\sum^{n}_{i=1}b_i}{n}$
$=\frac{n+(n-1)+\cdots+2+1}{n}\cdot\frac{b_1+b_2+\cdots+b_{n-1}+b_n}{n}$.
Fra Chebyshevs ulikhet har vi:
$\frac{n+(n-1)+\cdots+2+1}{n}\cdot\frac{b_1+b_2+\cdots+b_{n-1}+b_n}{n}\ge \frac{nb_1+(n-1)b_2+\cdots+2b_{n-1}+b_n}{n}$

Re: Ulikhetmaraton

Lagt inn: 26/01-2019 00:18
av Markus
En ganske lett en:

La $x,y \in \mathbb{R}$. Du kan anta at $\sin(x)$ er kontinuerlig. Vis at $$|\sin(x)-\sin(y)|<|x-y|$$

Re: Ulikhetmaraton

Lagt inn: 26/01-2019 01:09
av Aleks855
Jeg er ingen konkurransematematiker, men jeg begir meg ut på et forsøk. Og kan jeg anta at det menes $\leq$ siden $x = y$ er et moteksempel?

For $x = y$ er løsningen åpenbar.

Siden vi har absoluttverdi på begge sider kan vi anta WLOG at $x > y$.

La $f(a) = \sin(a)$ på $[y, x]$. Siden $f$ er kontinuerlig og deriverbar overalt (burde sistnevnte bevises?), kan vi vha. middelverdisetningen avgjøre at $\exists c \in (y, x) \ | \ f'(c) = \frac{f(x) - f(y)}{x-y}$.

Dette gir $$f'(c) = \cos(c) = \frac{\sin(x) - \sin(y)}{x-y} \ \Rightarrow \ \sin(x) - \sin(y) = \cos(c)(x-y) \ \Rightarrow \ |\sin(x) - \sin(y)| = |\cos(c)||x-y| \leq |x-y|$$ siden $\cos(c) \in [-1, 1] \ \Rightarrow \ |\cos(c)| \leq 1$

Ser nå at dette ikke er identisk med det som skulle vises, men det får være en øvelse til leseren å bytte om på $x$ og $y$ i beviset, som vil gi ønsket utfall :lol:

Siden jeg er ganske grønn på akkurat dette, så har jeg ingen go-to ressurser for å finne en passende oppfølger. Kanskje noen har en anbefaling på ei bok med ulikhetsnøtter av stigende vanskelighetsgrad?

Re: Ulikhetmaraton

Lagt inn: 26/01-2019 10:57
av Gustav
En liten omskrivning av beviset gitt av Aleks

Bevis ved motsigelse:

For $x=y$ holder ulikheten, så anta WLOG at $x>y$ og $|\frac{\sin x - \sin y}{x-y}|> 1$. Da fins en $c\in (y,x)$ slik at $|\cos c| = |(\sin c)'|=|\frac{\sin x - \sin y}{x-y}|> 1$, som er en motsigelse. Ergo følger utsagnet.


To lignende oppfølgere:

1. La $n\ge 2$ og $x_1,x_2,...,x_n$ være positive reelle tall. Vis at $$ \frac{1+x_1^2}{1+x_1x_2}+\frac{1+x_2^2}{1+x_2x_3}+\cdots +\frac{1+x_n^2}{1+x_nx_1}\ge n $$

2. La $n\ge 3$ og $a_1,a_2,..., a_n$ være positive reelle tall. Vis at $$ \frac{1+a_1^2}{a_2+a_3}+\frac{1+a_2^2}{a_3+a_4}+\cdots +\frac{1+a_n^2}{a_1+a_2}\ge n $$

Re: Ulikhetmaraton

Lagt inn: 26/01-2019 12:46
av zzzivert
En alternativ løsning til den trigonometriske ulikheten:

Vi bruker følgende:
1) $\sin(\alpha)-\sin(\beta)=2\sin(\frac{\alpha-\beta}{2})\cos(\frac{\alpha+\beta}{2})$
2) $|\cos(x)|\le 1$
3) $|\sin(x)|\le |x|$

I den opprinnelige ulikheten substituerer vi $x$ med $x+y$. Da får vi:
$|\sin(x+y)-\sin(y)|\le |x|$

Bruker vi 1), 2) og 3) får vi:
$|\sin(x+y)-\sin(y)|=2|\sin(\frac{x}{2})||\cos(\frac{2x+y}{2})|\le 2|\sin(\frac{x}{2})|\le2|\frac{x}{2}|=|x|$.